You are on page 1of 22

Chapter 8

Uncertainty and Risk


Exercise 8.1 Suppose you have to pay $2 for a ticket to enter a competition.
The prize is $19 and the probability that you win is 13 . You have an expected
utility function with u(x) = log x and your current wealth is $10.
1. What is the certainty equivalent of this competition?
2. What is the risk premium?
3. Should you enter the competition?
Outline Answer:
1. Given the probabilities and payos we have the following expected utility
if the person enters the lottery
Eu(x)

=
=
=

1
2
log (10 2 + 19) + log (10
3
3
1
2
log (27) + log (8)
3
3
log 3 + log 4 = log 12

2)

So the certainty equivalent is $12.


2. The expected wealth at the end of the period is
Ex =
=

1
2
[10 2 + 19] + [10
3
3
27 16
43
1
+
=
= 14
3
3
3
3

So the risk premium is $14 13

2]

$12 = $2 31 .

3. If the person does not enter the lottery he has just his initial wealth, $10.
So, in view of the answer to part (a) it makes sense to enter the lottery.

115

Microeconomics

CHAPTER 8. UNCERTAINTY AND RISK

Exercise 8.2 You are sending a package worth 10 000A


C. You estimate that
there is a 0.1 percent chance that the package will be lost or destroyed in transit. An insurance company o ers you insurance against this eventuality for a
premium of 15A
C. If you are risk-neutral, should you buy insurance?
Outline Answer:
No you should not. Your expected loss is 10 euros whereas the premium is
15 euros.

c Frank Cowell 2006

116

Microeconomics
Exercise 8.3 Consider the following denition of risk aversion. Let P :=
f(x! ; ! ) : ! 2 g be a random prospect, where x! is the payoPin state !
and ! is the (subjective) probability of state ! , and let Ex := !2 ! x! ,
the mean of the prospect, and let P := f( x! + [1
]Ex; ! ) : ! 2 g be a
mixture of the original prospect with the mean. Dene an individual as risk
averse if he always prefers P to P for 0 < < 1.
1. Illustrate this concept in (xred ; xblue )-space and contrast it with the concept
of risk aversion used in the text
2. Show that this denition of risk aversion need not imply convex-to-theorigin indi erence curves.
Outline Answer:
1. See Figure 8.1.
xBLUE

P
P
P

xRED

Figure 8.1: Nonconvex indierence curve


2. Let P be the prospect and P its mean. P can be any point in the line
joining them. The denition implies that moving along this line towards
P puts the person on a successively higher indierence curves. In Figure
8.1 it is clear that this condition is consistent with there being indierence
curves that violate the convex-to-the-origin property locally.

c Frank Cowell 2006

117

Microeconomics

CHAPTER 8. UNCERTAINTY AND RISK

Exercise 8.4 Suppose you are asked to choose between two lotteries. In one
case the choice is between P1 and P2 ;and in the other case the choice o ered is
between P3 and P4 , as specied below:
P1 :

$1; 000; 000 with probability 1


8
< $5; 000; 000 with probability 0.1
$1; 000; 000 with probability 0.89
P2 :
:
$0
with probability 0.01
P3 :

$5; 000; 000 with probability 0.1


$0
with probability 0.9

P4 :

$1; 000; 000


$0

with probability 0.11


with probability 0.89

It is often the case that people prefer P1 to P2 and then also prefer P3 to P4 .
Show that these preferences violate the independence axiom.
Outline Answer:
Let there be only three possible states of the world: red, blue and green,
with probabilities 0.01, 0.10, 0.89 respectively. Then the payos in the four
prospects can be written
P1
P2
P3
P4

red
1
0
0
1

blue
1
5
5
1

green
1
1
0
0

where all the entries in the table are in millions of dollars. Note that P1 and P2
have the same payo in the green state; P3 and P4 form a similar pair, except
that the payo in the green state is 0. Axiom 8.2 states that if P1 is preferred
to P2 than any other similar pair of prospects (1; 1; z) and (0; 5; z) ought also
to be ranked in the same order, for arbitrary z: but this would imply that P4
is preferred to P3 , the opposite of the preferences as stated.
Note also that if the preferences had been such that P4 was preferred to P3
then the independence axiom would imply that P1 was preferred to P2 .

c Frank Cowell 2006

118

Microeconomics
Exercise 8.5 This is an example to illustrate disappointment. Suppose the
payo s are as follows
x00 weekend for two in your favourite holiday location
x0 book of photographs of the same location
x
sh-and-chip supper
Your preferences under certainty are x00 x0 x . Now consider the following
two prospects
8 00
with probability 0:99
< x
x0 with probability 0
P1 :
:
x with probability 0:01
8 00
with probability 0:99
< x
x0 with probability 0:01
P2 :
:
x with probability 0

Suppose a person expresses a preference for P1 over P2 . Briey explain why


this might be the case in practice. Which of the three axioms State Irrelevance,
Independence, Revealed Likelihood, is violated by such preferences?
Outline Answer:
It is possible that, given the information that the rst event (with payo x00 )
has not happened you would then prefer x to x0 : photographs of your favourite
holiday spot may be too painful once you know that the holiday is not going to
happen. So you may prefer P1 over P2 .
These preferences violate the independence axiom. To see this, note that,
by the revealed likelihood axiom, since x0 is strictly preferred to x , it must be
the case that P20 is strictly preferred to P10 , where
P10 :

x0
x

with probability 0
with probability 1

P20 :

x0
x

with probability 0:01


with probability 0:99

But P10 and P20 can be written equivalently as


8
< x with probability
x0 with probability
P10 :
:
x with probability
8
< x with probability
x0 with probability
P20 :
:
x with probability

0:99
0
0:01
0:99
0:01
0

By the independence axiom if P20 is strictly preferred to P10 , then P2 must be


strictly preferred to P1 .

c Frank Cowell 2006

119

Microeconomics

CHAPTER 8. UNCERTAINTY AND RISK

Exercise 8.6 An example to illustrate regret. Let


P := f(x! ;

!)

:!2

P 0 := f(x0! ;

!)

:!2 g

be two prospects available to an individual. Dene the expected regret if the


person chooses P rather than P 0 as
X
0
x! ; 0g
(8.1)
! max fx!
!2

Now consider the choices amongst prospects presented in Exercise 8.4. Show
that if a person is concerned to minimise expected regret as measured by (8.1),
then it is reasonable that the person select P2 when P1 is also available and then
also select P4 when P3 is available.
Outline Answer:
Denote the regret in (8.1) by r (P; P 0 ).
If I choose P2 when P1 is also available then the regret is
r (P2 ; P1 )

0:1 [0] + 0:89 [0] + :01 [1]

10; 000

Whereas, had I chosen P1 when P2 was available, then the regret would have
been
r (P1 ; P2 )

0:1 [4; 000; 000] + 0:89 [0] + :01 [0]

400; 000

If I choose P4 when P3 is also available then the regret is


r (P4 ; P3 )

0:1 [0] + 0:89 [0] + :01 [0]

Whereas, had I chosen P3 when P4 was available, then the regret would have
been
r (P3 ; P4 )

c Frank Cowell 2006

0:1 [0] + 0:89 [0] + :01 [5; 000; 000]

50; 000

120

Microeconomics
Exercise 8.7 An example of the Ellsberg paradox . There are two urns marked
Left and Right each of which contains 100 balls. You know that in Urn L
there exactly 49 white balls and the rest are black and that in Urn R there are
black and white balls, but in unknown proportions. Consider the following two
experiments:
1. One ball is to be drawn from each of L and R. The person must choose
between L and R before the draw is made. If the ball drawn from the chosen
urn is black there is a prize of $1000, otherwise nothing.
2. Again one ball is to be drawn from each of L and R; again the person must
choose between L and R before the draw. Now if the ball drawn from the
chosen urn is white there is a prize of $1000, otherwise nothing.
You observe a person choose Urn L in both experiments. Show that this
violates the Revealed Likelihood Axiom.

Outline Answer
The implication of the revealed likelihood axiom is that there exist subjective
probabilities ! . The result is proved by showing that it the stated behaviour
is inconsistent with the existence of subjective probabilities.
In this case the revealed likelihood axiom implies that for each urn there is
a given subjective probability of drawing a black ball L (left-hand urn) and R
(right-hand urn) such that preferences can be represented as
vblack (xblack ; xwhite ) + [1

] vwhite (xblack ; xwhite )

(8.2)

where = L or R and xblack and xwhite are the payos if a black ball or a
white ball are drawn respectively.
Note that the representation (8.2) does not impose either the State Irrelevance Axiom (which would require that vblack ( ) and vwhite ( ) be the same
function) or the Independence axiom (which would require that vblack ( ) be a
function only of xblack etc.). Nor does it impose the common-sense requirement
that L = 0:49. All we need below is the very weak assumption that preferences
are not perverse:
vblack (1000; 0) > vwhite (1000; 0)
(8.3)
and
vblack (0; 1000) < vwhite (0; 1000)

(8.4)

Condition (8.3) simply says that if the $1000 prize is attached to a black ball
then the utility to be derived from having selected a black ball is higher than
selecting a white ball; condition (8.4) is the counterpart when the prize attaches
to the white ball..
Experiment 1 suggests that

>
c Frank Cowell 2006

L vblack

(1000; 0) + [1

L ] vwhite

(1000; 0)

R vblack

(1000; 0) + [1

R ] vwhite

(1000; 0)

121

(8.5)

Microeconomics

CHAPTER 8. UNCERTAINTY AND RISK

while experiment 2 suggests that


L vblack

>

(0; 1000) + [1

L ] vwhite

R vblack (0; 1000) + [1

(0; 1000)

R ] vwhite (0; 1000)

(8.6)

We can see that (8.5) implies


L

[vblack (1000; 0)

vwhite (1000; 0)] >

from which we deduce that, given (8.3),


L

[vblack (0; 1000)

vwhite (0; 1000)] >

[vblack (1000; 0)

>

[vblack (0; 1000)

R.

vwhite (1000; 0)]

However (8.5) implies


vwhite (0; 1000)] :

So that, given (8.4), we would have L < R a contradiction. Therefore the


revealed likelihood axiom must be violated.

c Frank Cowell 2006

122

Microeconomics
Exercise 8.8 An individual faces a prospect with a monetary payo represented
by a random variable x that is distributed over the bounded interval of the real
line [a; a]. He has a utility function Eu(x) where
u(x) = a0 + a1 x

1
a2 x2
2

and a0 ; a1 ; a2 are all positive numbers.


1. Show that the individuals utility function can also be written as '(Ex; var(x)).
Sketch the indi erence curves in a diagram with Ex and var(x) on the
axes, and discuss the e ect on the indi erence map altering (i) the parameter a1 , (ii) the parameter a2 .
2. For the model to make sense, what value must a have? [Hint: examine
the rst derivative of u.]
3. Show that both absolute and relative risk aversion increase with x.
Outline Answer:
1. Clearly
1
Eu(x) = a0 + a1 E(x) + a2 [(E(x))2
2
Marginal utility is a1

var(x)]:

a2 x:

2. For this to be non-negative we must have E(x)


a1 =a2 hence the indierence curves are depicted with E(x) as good, var(x) as bad and
M RS = 2 [a1 =a2 E(x)] :
3.

where and 0

c Frank Cowell 2006

ux (x)

a1 + a2 x

uxx (x)

a2

(x)

(x)

%(x)

xmax :=

uxx (x)
=
ux (x)
1
xmax x
1
xmax =x 1
a1
a2 :

123

a2
a1 + a2 x

Microeconomics

CHAPTER 8. UNCERTAINTY AND RISK

Exercise 8.9 A person lives for 1 or 2 periods. If he lives for both periods he
has a utility function given by
U (x1 ; x2 ) = u (x1 ) + u (x2 )

(8.7)

where the parameter is the pure rate of time preference. The probability of
survival to period 2 is , and the persons utility in period 2 if he does not
survive is 0.
1. Show that if the persons preferences in the face of uncertainty are represented by the expected-utility functional form
X
(8.8)
! u (x! )
!2

then the persons utility can be written as


u (x1 ) + 0 u (x2 ) :
What is the value of the parameter

(8.9)

2. What is the appropriate form of the utility function if the person could live
for an indenite number of periods, the rate of time preference is the same
for any adjacent pair of periods, and the probability of survival to the next
period given survival to the current period remains constant?

Outline Answer:
1. Consider the persons lifetime utility with the consumption x1 and x2
in the two periods. If the person survives into the second period utility
is given by u (x1 ) + u (x2 ) otherwise it is just u (x1 ). Given that the
probability of the event survive to second period is expected lifetime
utility is
[u (x1 ) + u (x2 )] + [1
] u (x1 ) :
On rearranging we get
u (x1 ) +
in other words the form (8.9) with

u (x2 ) ;
0

(8.10)

2. Apply the argument to one more period. Now there is consumption


x1 ,x2 ; x3 in the three periods and the probability of surviving into period
t + 1 given that you have made it to period t is still . Consider the
situation of someone who survives to period 2. The person gets utility
u (x2 ) + u (x3 )

(8.11)

if he survives to period 3 and u (x2 ) otherwise. His expected utility for


the rest of his lifetime, contingent on having reached period 2 is therefore
[u (x2 ) + u (x3 )] + [1
= u (x2 ) +
c Frank Cowell 2006

u (x3 )
124

] u (x2 )
(8.12)

Microeconomics
So now view the situation from the position of the beginning of the lifetime.
The person gets utility
u (x1 ) + [u (x2 ) +

u (x3 )]

(8.13)

if he makes it through to period 2, where the expression in square brackets


in (8.13) is just the rest-of-lifetime expected utility if you get to period 2,
taken from (8.12); of course if the person does not survive period 1 he gets
just u (x1 ). So, using the same reasoning as before, from the standpoint
of period 1 lifetime expected utility is now
[u (x1 ) + [u (x2 ) +
+ [1

u (x3 )]]

] u (x1 ) :

Rearranging this we have


u (x1 ) +

u (x2 ) +

2 2

u (x2 ) :

(8.14)

It is clear that the same argument could be applied to T > 2 periods and
that the resulting utility function would be of the form
u (x1 ) +

u (x2 ) +

2 2

u (x2 ) + ::: +

T T

u (x2 ) :

(8.15)

In other words we have the standard intertemporal utility function with


the pure rate of time preference replaced by the modied rate of time
preference 0 := .

c Frank Cowell 2006

125

Microeconomics

CHAPTER 8. UNCERTAINTY AND RISK

Exercise 8.10 A person has an objective function Eu(y) where u is an increasing, strictly concave, twice-di erentiable function, and y is the monetary value
of his nal wealth after tax. He has an initial stock of assets K which he may
keep either in the form of bonds, where they earn a return at a stochastic rate
r, or in the form of cash where they earn a return of zero. Assume that Er > 0
and that Prfr < 0g > 0.
1. If he invests an amount in bonds (0 < < K) and is taxed at rate t
on his income, write down the expression for his disposable nal wealth y,
assuming full loss o set of the tax.
2. Find the rst-order condition which determines his optimal bond portfolio
.
3. Examine the way in which a small increase in t will a ect

4. What would be the e ect of basing the tax on the persons wealth rather
than income?

Outline Answer:
1. Suppose the person puts an amount
in bonds leaving the remaining
K
of assets in cash. Then, given that the rate of return on cash is
zero and on bonds is the stochastic variable r, income is
[K

]0 + r = r

If the tax rate is t then, given that full loss oset implies that losses and
gains are treated symmetrically, disposable income is
[1

t] r

and (disposable) nal wealth is


x =

[K

]+

+ [1
[value of b onds]

[cash]

K + [1

t] r

[incom e]

t] r:

(8.16)

Note that x is a stochastic variable and could be greater or less than initial
wealth K.
2. The individuals optimisation problem is to choose
Using (8.16) the FOC for an interior solution is
E (ux (x) [1

to maximise Eu(x).

t] r) = 0;

which implies
E (ux (x)r) = 0:

(8.17)

Solving this determines


=
(t; K), the optimal bond purchases that
depends on the tax rate and initial wealth as well as the distribution of
returns and risk aversion.
c Frank Cowell 2006

126

Microeconomics
3. Take the FOC (8.17). Substituting for x from (8.16) and dierentiating
with respect to t we get
E uxx (x)

@
[1
@t

r+

E uxx (x)r2

t] r r

@
[1
@t

= 0;

t]

= 0:

so that
+

@
[1
@t

t]
@
@t

An increase in the tax rate increases the demand for bonds.


4. Final wealth is initial wealth plus income. If the tax is on wealth then
disposable nal wealth is
x = [1

t] K + [1

t] r

(8.18)

instead of (8.16). Clearly the FOC (8.17) remains essentially unaltered


(the new tax just reduces total wealth). Dierentiating the FOC with x
dened by (8.18) we now nd
E uxx (x)

r+

@
[1
@t

KE (uxx (x)r) + E uxx (x)r2

t] r r

= 0;

@
[1
@t

t]

= 0:

This implies
K

E (uxx (x)r)
@
+
[1
2
E (uxx (x)r )
@t

@
[1
@t
@
=
@t
1

t] =

+K
+

K
1

t] = 0:

E (uxx (x)r)
:
E (uxx (x)r2 )

E (uxx (x)r)
:
t E (uxx (x)r2 )

The rst term on the right-hand side is positive; as for the second term,
the denominator is negative and the numerator is positive, given DARA.
So the impact of tax on bond-holding is now ambiguous.

c Frank Cowell 2006

127

Microeconomics

CHAPTER 8. UNCERTAINTY AND RISK

Exercise 8.11 An individual taxpayer has an income y that he should report to


the tax authority. Tax is payable at a constant proportionate rate t. The taxpayer
reports x where 0 x y and is aware that the tax authority audits some tax
returns. Assume that the probability that the taxpayers report is audited is
, that when an audit is carried out the true taxable income becomes public
knowledge and that, if x < y, the taxpayer must pay both the underpaid tax and
a surcharge of s times the underpaid tax.
1. If the taxpayer chooses x < y, show that disposable income c in the two
possible states-of-the-world is given by
cnoaudit
caudit

= y

tx;

[1

st] y + stx:

2. Assume that the individual chooses x so as to maximise the utility function


[1

] u (cnoaudit ) + u (caudit ) :

where u is increasing and strictly concave.


(a) Write down the FOC for an interior maximum.
(b) Show that if 1
s > 0 then the individual will denitely underreport income.
3. If the optimal income report x satises 0 < x < y:
(a) Show that if the surcharge is raised then under-reported income will
decrease.
(b) If true income increases will under-reported income increase or decrease?
Outline Answer:
If the individual reports x then he pays tax tx i.e. he underpays an amount
t [y x]. So
1. If the under-reporting remains undetected then
cnoaudit

tx

ty + t [y

x]

and if the audit takes place then


caudit

tx

[1 + s] t [y

[1

st] y + stx

x]

2. The individual maximises


Eu(c) = [1

] u (y

tx) + u ([1

st] y + stx)

Dierentiating this we have


@Eu(c)
=
@x

t [1

] uc (y

tx) + st uc ([1

where uc ( ) denotes the rst derivative of u.


c Frank Cowell 2006

128

st] y + stx)

Microeconomics
(a) If there is an interior maximum at x then the following FOC must
hold
[1
] uc (y tx ) = s uc ([1 t st] y + stx ) :
(b) If the person reports fully then
@Eu(c)
@x

t [1

[1

] uc (y

ty) + st uc ([1

t] y)

x=y

s ] tuc (y

ty)

Given that t and uc are positive it is clear that the above expression
is negative if 1
s > 0. Therefore the individuals expected
utility would increase if he reduced x below y.
3. Dierentiating the FOC with respect to s and rearranging we get

@x
@x
s2 t ucc ([1 t st] y + stx )
@s
@s
st] y + stx ) + st [x
y] ucc ([1 t st] y + stx )

t [1

] ucc (y

uc ([1

tx )

(a) Therefore
uc (caudit ) + st [x
t

@x
=
@s

y] ucc (caudit )

(8.19)

where
:=

[1

] ucc (y

tx )

s2 ucc ([1

st] y + stx ) > 0

Given that uc > 0, x < y and ucc < 0 it is clear that the numerator
of (8.19) is positive.x increases with s so t [y x ] decreases.
(b) Dierentiating the FOC with respect to y we get
[1
=

] ucc (y

s ucc ([1

tx ) 1

@x
@y

st] y + stx ) [1

st] + st

@x
@y

Therefore we have
@x
=
@y

s [1

t
t [[1

st] ucc (caudit ) [1


] ucc (cnoaudit )
] ucc (cnoaudit ) + s2 ucc (caudit )]

@ [y x ]
s [1 t st] ucc (caudit ) [1
] ucc (cnoaudit )
=1+
@y
[[1
] tucc (cnoaudit ) + s2 tucc (caudit )]
@ [y x ]
=
@y

t [1
t [[1

] ucc (cnoaudit )
succ (caudit )
] ucc (cnoaudit ) + s2 ucc (caudit )]

This is of ambiguous sign unless we assume DARA in which case it


is positive.

c Frank Cowell 2006

129

Microeconomics

CHAPTER 8. UNCERTAINTY AND RISK

Exercise 8.12 A risk-averse person has wealth y0 and faces a risk of loss
L < y0 with probability . An insurance company o ers cover of the loss at
a premium > L. It is possible to take out partial cover on a pro-rata basis,
so that an amount tL of the loss can be covered at cost t where 0 < t < 1.
1. Explain why the person will not choose full insurance
2. Find the conditions that will determine t , the optimal value of t.
3. Show how t will change as y0 increases if all other parameters remain
unchanged.
Outline Answer:
1. Consider the persons wealth after taking out (partial) insurance cover
using the two-state model (no loss;loss). If the person remained uninsured it would be (y0 ; y0 L); if he insures fully it is (y0
; y0
). So
if he insures a proportion t for the pro-rata premium wealth in the two
states will be
([1

t] y0 + t [y0

] ; [1

t] [y0

L] + t [y0

])

which becomes
(y0

t ; y0

[1

t] L)

So expected utility is given by


Eu = [1

] u (y0

t ) + u (y0

[1

t] L)

Therefore
@Eu
=
@t

[1

] uy (y0

t ) + [L

] uy (y0

[1

t] L)

Consider what happens in the neighbourhood of t = 1 (full insurance).


We get
@Eu
@t

[1

] uy (y0

) + [L

] uy (y0

t=1

[L

] uy (y0

We know that uy (y0


) > 0 (positive marginal utility of wealth) and, by
assumption, L < . Therefore this expression is strictly negative which
means that in the neighbourhood of full insurance (t = 1) the individual
could increase expected utility by cutting down on the insurance cover.
2. For an interior maximum we have
@Eu
=0
@t
which means that the optimal t is given as the solution to the equation
[1

] uy (y0

c Frank Cowell 2006

) + [L
130

] uy (y0

[1

t ] L) = 0

Microeconomics
3. Dierentiating the above equation with respect to y0 we get
[1

] uyy (y0

@t
+[L
@y0

) 1

] uyy (y0

[1

t ] L) 1

which gives
[1
@t
=
@y0
[1

] uyy (y0
] uyy (y0

t
t

)
2

[L
+ [L

] uyy (y0
]

uyy (y0

[1
t

[1

t ] L)
t ] L)

The denominator of this must be negative: uyy ( ) is everywhere negative


and the other terms are positive. The numerator is positive if DARA
holds: therefore an increase in wealth reduces the demand for insurance
coverage.

c Frank Cowell 2006

131

L]

@t
=0
@y0

Microeconomics

CHAPTER 8. UNCERTAINTY AND RISK

Exercise 8.13 Consider a competitive, price-taking rm that confronts one of


the following two situations:
uncertainty: price p is a random variable with expectation p.
certainty: price is xed at p.
It has a cost function C(q) where q is output and it seeks to maximise the
expected utility of prot.
1. Suppose that the rm must choose the level of output before the particular
realisation of p is announced. Set up the rms optimisation problem and
derive the rst- and second-order conditions for a maximum. Show that,
if the rm is risk averse, then increasing marginal cost is not a necessary
condition for a maximum, and that it strictly prefers certainty to uncertainty. Show that if the rm is risk neutral then the rm is indi erent
as between certainty and uncertainty.
2. Now suppose that the rm can select q after the realisation of p is announced, and that marginal cost is strictly increasing. Using the rms
competitive supply function write down prot as a function of p and show
that this prot function is convex. Hence show that a risk-neutral rm
would strictly prefer uncertainty to certainty.
Outline Answer:
1. Prot is given by
:= pq

C(q)

where p is a random variable. Maximising expected utility of prot Eu( )


by choice of q requires the FOC
E(u ( )p)

E(u ( ))Cq = 0

where u ( ) is the rst derivative of u( ). This will represent a maximum


if
d2 Eu
< 0:
dq 2
We nd that this implies
E(u

[p

Cq ]2 )

E(u

)Cqq < 0:

Notice that since the rst term is negative for a risk-averse rm then the
condition can be satised not only if Cqq > 0 but also if Cqq < 0 and jCqq j
is not too large. Now consider transforming p to pb thus: pb = (1
)p + p
then pb has the same mean as p but is less dispersed. Maximised utility for
the random variable pb is
Eu([(1

)p + p]q

C(q ))

where q is the output satisfying the rst-order conditions for a maximum.


Dierentiate this expected utility with respect to
@Eu
= [E(u [p
@
c Frank Cowell 2006

p])]q + [E(u [b
p
132

Cq ])]

@q
@

Microeconomics
where the last term vanishes because of the rst order condition. So
@Eu
has the sign of E(u [p p]): But this must be positive if u is
@
decreasing with and will be zero if u is constant. Hence the rm strictly
prefers certainty if it is risk averse and is indierent between certainty and
uncertainty if it is risk neutral.
2. For any known realization p we may write q = S(p) where S is the competitive rm supply curve. Prots as a function of P may thus be written:
(p) = pS(p)

C(S(p))

which implies
d (p)
= [p
dp

Cq ]Sp (p) + S(p) = S(p)

(8.20)

where Sp (p) is the slope of the supply curve at p, a positive number.


Therefore, dierentiating (8.20) we have
d2 (p)
= Sp (p) > 0:
dp2
Hence ( ) is increasing and convex. So it is immediate that E (p) >

c Frank Cowell 2006

133

(p):

Microeconomics

CHAPTER 8. UNCERTAINTY AND RISK

Exercise 8.14 Every year Alf sells apples from his orchard. Although the market price of apples remains constant (and equal to 1), the output of Alf s orchard
is variable yielding an amount R1 ; R2 in good and poor years respectively; the
probability of good and poor years is known to be 1
and respectively. A
buyer, Bill, o ers Alf a contract for his apple crop which stipulates a down payment (irrespective of whether the year is good or poor) and a bonus if the year
turns out to be good.
1. Assuming Alf is risk averse, use an Edgeworth box diagram to sketch the
set of such contracts which he would be prepared to accept. Assuming that
Bill is also risk averse, sketch his indi erence curves in the same diagram.
2. Assuming that Bill knows the shape of Alf s acceptance set, illustrate the
optimum contract on the diagram. Write down the rst-order conditions
for this in terms of Alf s and Bills utility functions.

xRED

a
xBLUE

R2

b
xBLUE

R1

xRED

Figure 8.2: Acceptable contracts


Outline Answer:
1. In Figure 8.2 the contours represent Alfs indierence curves: note that
they are convex to the point 0a (risk aversion) and that they have the same
slope [1
] = where they cross the 45 ray through 0a (consequence of
von-Neumann utility function). Point D represents the initial endowment;
Alfs endowment is (R1 ; R2 ). Alfs indierence curve through point D
represents the boundary of the set of consumptions that Alf would regard
as being at least as good as the initial endowment: the shaded area is his
acceptance set. The buyer (Bill) has an endowment K that is independent
of the state of the world see Figure 8.3. Note that the indierence curves
for Bill also have the slope [1
] = where they cross the 45 ray through
0b :
2. Point E in Figure ?? represents the optimum contract (from Bills point
of view) since it is a point of common tangency of two indierence curves.
c Frank Cowell 2006

134

Microeconomics
b

xRED

a
xBLUE

b
xBLUE

xRED

Figure 8.3: Buyers situation


b

xRED

a
xBLUE

R2

b
xBLUE

R1

Figure 8.4: Optimal contract


At E we have that

c Frank Cowell 2006

ua0 (xa1 )
ub0 (xb )
= b0 1b :
a
a0
u (x2 )
u (x2 )

135

xRED

Microeconomics

CHAPTER 8. UNCERTAINTY AND RISK

Exercise 8.15 In exercise 8.14, what would be the e ect on the contract if (i)
Bill were risk neutral; (ii) Alf risk neutral?
Outline Answer:
In case (i) Bills indierence curves become lines with slope [1
] = and
the optimum is at E in Figure 8.5. In case (ii) Alfs indierence curves become
lines with slope [1
] = and the optimum is at the endowment point D.
b

xRED

a
xBLUE

R2

b
xBLUE

R1

Figure 8.5: Optimal contract: risk-neutral buyer

c Frank Cowell 2006

136

You might also like